AIME 2015 Problem 13

Algebra Level 5

Define the sequence a 1 , a 2 , a 3 , a_1, a_2, a_3, \ldots by a n = k = 1 n sin k a_n = \sum\limits_{k=1}^n \sin{k} , where k k represents radian measure. Find the index of the 100 100 th term for which a n < 0 a_n < 0 .


Try more problems of AIME from this set AIME 2015


The answer is 628.

This section requires Javascript.
You are seeing this because something didn't load right. We suggest you, (a) try refreshing the page, (b) enabling javascript if it is disabled on your browser and, finally, (c) loading the non-javascript version of this page . We're sorry about the hassle.

2 solutions

Mamunur Rashid
Jan 18, 2016

De Moiver's formula is widely used to calculate the summation however it is sometimes also calculated by using Telescopic summation method. Here its a version of the calculation using the telescopic method. a n = k = 1 n sin k = 1 2 csc ( 1 2 ) k = 1 n ( 2 s i n ( 1 2 ) sin ( 2 k 2 ) ) = 1 2 csc ( 1 2 ) k = 1 n ( cos ( 2 k 1 2 ) cos ( 2 k + 1 2 ) ) \begin{gathered} {a_n} = \sum\limits_{k = 1}^n {\sin k} \\ {\text{ }} = \frac{1}{2}\csc \left( {\frac{1}{2}} \right)\sum\limits_{k = 1}^n {\left( {2sin\left( {\frac{1}{2}} \right)\sin \left( {\frac{{2k}}{2}} \right)} \right)} \\ {\text{ }} = \frac{1}{2}\csc \left( {\frac{1}{2}} \right)\sum\limits_{k = 1}^n {\left( {\cos \left( {\frac{{2k - 1}}{2}} \right) - \cos \left( {\frac{{2k + 1}}{2}} \right)} \right)} \\ \end{gathered} = 1 2 csc ( 1 2 ) ( cos ( 1 2 ) + k = 2 n cos ( 2 k 1 2 ) = k = 1 n 1 cos ( 2 k + 1 2 ) k = 1 n 1 cos ( 2 k + 1 2 ) cos ( 2 n + 1 2 ) ) = 1 2 csc ( 1 2 ) ( cos ( 1 2 ) cos ( 2 n + 1 2 ) + k = 1 n 1 cos ( 2 k + 1 2 ) k = 1 n 1 cos ( 2 k + 1 2 ) ) = 1 2 csc ( 1 2 ) ( cos ( 1 2 ) cos ( 2 n + 1 2 ) ) \begin{gathered} = \frac{1}{2}\csc \left( {\frac{1}{2}} \right)\left( {\cos \left( {\frac{1}{2}} \right) + \underbrace {\sum\limits_{k = 2}^n {\cos \left( {\frac{{2k - 1}}{2}} \right)} }_{ = \sum\limits_{k = 1}^{n - 1} {\cos \left( {\frac{{2k + 1}}{2}} \right)} } - \sum\limits_{k = 1}^{n - 1} {\cos \left( {\frac{{2k + 1}}{2}} \right)} - \cos \left( {\frac{{2n + 1}}{2}} \right)} \right){\text{ }} \\ = {\text{ }}\frac{1}{2}\csc \left( {\frac{1}{2}} \right)\left( {\cos \left( {\frac{1}{2}} \right) - \cos \left( {\frac{{2n + 1}}{2}} \right) + \sum\limits_{k = 1}^{n - 1} {\cos \left( {\frac{{2k + 1}}{2}} \right) - \sum\limits_{k = 1}^{n - 1} {\cos \left( {\frac{{2k + 1}}{2}} \right)} } } \right){\text{ }} \\ {\text{ = }}\frac{1}{2}\csc \left( {\frac{1}{2}} \right)\left( {\cos \left( {\frac{1}{2}} \right) - \cos \left( {\frac{{2n + 1}}{2}} \right)} \right) \\ \end{gathered} now, a n < 0 implies 1 2 csc ( 1 2 ) ( cos ( 1 2 ) cos ( 2 n + 1 2 ) ) < 0 {\text{now, }}{a_n} < 0{\text{ implies }}\frac{1}{2}\csc \left( {\frac{1}{2}} \right)\left( {\cos \left( {\frac{1}{2}} \right) - \cos \left( {\frac{{2n + 1}}{2}} \right)} \right) < 0 cos ( 2 n + 1 2 ) > cos ( 1 2 ) 2co s 2 ( 2 n + 1 4 ) 1 > 2co s 2 ( 1 4 ) 1 co s 2 ( 2 n + 1 4 ) > co s 2 ( 1 4 ) sec 2 ( 2 n + 1 4 ) < sec 2 ( 1 4 ) 1 + tan 2 ( 2 n + 1 4 ) < 1 + tan 2 ( 1 4 ) tan 2 ( 2 n + 1 4 ) < tan 2 ( 1 4 ) tan ( 2 n + 1 4 ) < tan ( 1 4 ) tan ( 1 4 ) < tan ( 2 n + 1 4 ) < tan ( 1 4 ) \begin{gathered} \Rightarrow \cos \left( {\frac{{2n + 1}}{2}} \right) > \cos \left( {\frac{1}{2}} \right){\text{ }} \\ \Rightarrow {\text{2co}}{{\text{s}}^2}\left( {\frac{{2n + 1}}{4}} \right) - 1 > {\text{2co}}{{\text{s}}^2}\left( {\frac{1}{4}} \right) - 1{\text{ }} \\ \Rightarrow {\text{co}}{{\text{s}}^2}\left( {\frac{{2n + 1}}{4}} \right) > {\text{co}}{{\text{s}}^2}\left( {\frac{1}{4}} \right){\text{ }} \\ \Rightarrow {\sec ^2}\left( {\frac{{2n + 1}}{4}} \right) < {\sec ^2}\left( {\frac{1}{4}} \right) \\ \Rightarrow 1 + {\tan ^2}\left( {\frac{{2n + 1}}{4}} \right) < 1 + {\tan ^2}\left( {\frac{1}{4}} \right){\text{ }} \\ \Rightarrow {\tan ^2}\left( {\frac{{2n + 1}}{4}} \right) < {\tan ^2}\left( {\frac{1}{4}} \right){\text{ }} \\ \Rightarrow \left| {\tan \left( {\frac{{2n + 1}}{4}} \right)} \right| < {\text{ }}\tan \left( {\frac{1}{4}} \right) \\ \Rightarrow - \tan \left( {\frac{1}{4}} \right) < \tan \left( {\frac{{2n + 1}}{4}} \right) < \tan \left( {\frac{1}{4}} \right) \\ \end{gathered} tan ( m π 1 4 ) < tan ( 2 n + 1 4 ) < tan ( m π + 1 4 ) , becasue tan ( x ) tan ( m π + x ) for all m Z m π 1 4 < 2 n + 1 4 < m π + 1 4 2 π m 1 < n < 2 π m n ( 2 π m 1 , 2 π m ) \begin{gathered} \Rightarrow \tan \left( {m\pi - \frac{1}{4}} \right) < \tan \left( {\frac{{2n + 1}}{4}} \right) < \tan \left( {m\pi + \frac{1}{4}} \right),{\text{ becasue tan}}\left( x \right) \equiv \tan \left( {m\pi + x} \right){\text{ for all }}m \in \mathbb{Z} \\ \Rightarrow m\pi - \frac{1}{4} < \frac{{2n + 1}}{4} < m\pi + \frac{1}{4} \\ {\text{ }} \Rightarrow {\text{ }}2\pi m - 1 < n < 2\pi m \\ {\text{ }} \Rightarrow n \in (2\pi m - 1,2\pi m) \\ \end{gathered} Here,m represents the number of first m cyclies of the continuous function of 1 2 csc ( 1 2 ) ( cos ( 1 2 ) cos ( 2 n + 1 2 ) ) and also the numer of first m negative terms of the sequence because there exists only 1 negative value of a n in each [ 2 π ( m 1 ) , 2 π m ] Now,the length of the interval (2m π - 1,2m π ) is 1 indicating that only one integer value for n is possible in this interval . For m = 100 , n ( 627.31... , 628.31... ) and since n Z , n = 628 \begin{gathered} {\text{Here,m represents the number of first m cyclies of }} \\ {\text{the continuous function of }}\frac{1}{2}\csc \left( {\frac{1}{2}} \right)\left( {\cos \left( {\frac{1}{2}} \right) - \cos \left( {\frac{{2n + 1}}{2}} \right)} \right){\text{ }} \\ {\text{and also the numer of first m negative terms of the sequence because }} \\ {\text{there exists only 1 negative value of }}{a_n}{\text{ in each [}}2\pi (m - 1),2\pi m] \\ {\text{Now,the length of the interval (2m}}\pi {\text{ - 1,2m}}\pi {\text{) is 1 indicating that }} \\ {\text{only one integer value for n is possible in this interval }}{\text{.}} \\ {\text{For }}m = 100,{\text{ }}n \in (627.31...,628.31...){\text{ and since }}n \in \mathbb{Z},{\text{ }}n = \boxed{628}{\text{ }} \\ \end{gathered}

finally able to solve it

Mardokay Mosazghi - 5 years, 1 month ago

Log in to reply

I'm really sorry for making it too lengthy. I just tried to include all the steps in details for everyone's understanding.

MAMUNUR RASHID - 5 years, 1 month ago

Log in to reply

your solution is nice

Mardokay Mosazghi - 5 years, 1 month ago

Log in to reply

@Mardokay Mosazghi Thank you.

MAMUNUR RASHID - 5 years, 1 month ago

2 π = 44 7 . S o o n c e i n 2 π , t h e r e w i l l b e o n e t i v e . 2*\pi=\dfrac{44} 7. \ \ So\ once \ in \ 2*\pi, there \ will \ be \ one \ -tive.\\
So in 44 radians, there would be 7, negatives.
For any cycle, the index at the start of 7 step cycle is 6 * 6+1=37 less that the end index.
Next cycle starts after 7 more indices. So between cycles the similarly located indices are 44 apart.
From the first index of the cycle, next is at 6 more indices, till third.
Between 3rd and four there are 7 more indices.
From the forth index of the cycle, next is at 6 more indices, till the last-7th.
First cycle starts with first negative at index 6. Thus the 7th negative is at index 43.



So to have 98 negative, we need 14 * 44-1=615 as index.
For next cycle 7 more. That is for 99 negative, the index is 615+7=622.\
For 100 negatives, the index is 622+6= 628. \Large\ \ \ \color{#D61F06}{628}.
In short negative appear as follows.
At (6-12-18--25-31-37-43)--(50-56-62--69-75-81-87)--(94 to 131).......(572 to 615)--622-628
In a cycle first at n, then n+6, n+12, n+19, n+25, n=31, n+37
So 7 negatives at 43,..... 14 at 43+44=87,..... 21 at 87+44=131,..... 7n at n * 44-1.
The diffrance 2 π 44 7 = . 002528 2*\pi-\dfrac{44} 7=-.002528 would have effect for very large indices
and there will have to be a correction from the above method.

\text{The index f(x) for x appearances of negative value, for x<2251 is as under.}\\ \color{#3D99F6}{f(x)=\ \left \lfloo \dfrac{44*x} 7 \rigrt \rfloor} \ \ \ \ -\ \left \lfloor \dfrac x {161} \ \rigrt \rfloor}\ \ \left \lfloor \dfrac x {511} \ \rigrt \rfloor}\ -\ \left \lfloor \dfrac x {970} \ \rigrt \rfloor}\ -\ \left \lfloor \dfrac x {1355} \ \rigrt \rfloor}\ -\ \left \lfloor \dfrac x {1665} \ \rigrt \rfloor}\ }

0 pending reports

×

Problem Loading...

Note Loading...

Set Loading...